LSAT and Law School Admissions Forum

Get expert LSAT preparation and law school admissions advice from PowerScore Test Preparation.

 Administrator
PowerScore Staff
  • PowerScore Staff
  • Posts: 8917
  • Joined: Feb 02, 2011
|
#25854
Complete Question Explanation

Flaw in the Reasoning. The correct answer choice is (B)

This question provides a great example of why the length of a stimulus does not necessarily correlate to the difficulty of the argument contained within it. The question stem tells you that this is a Flaw question, and focuses you in on the flaw in the astronomer’s argument. While LSAC used many words to slow you down, the astronomer essentially argues that Professor Tagar cannot be correct because he does not accept the view of two other biologists. The astronomer’s sole premise is that “Tagar does not accept the views of biologists Swiderski and Terrada, who maintain that Martian bacteria would shrink to one-tenth of 1 percent of their normal volume when water or other nutrients were in short supply.” The astronomer provides no other support for the conclusion that “Tagar’s view cannot be right.”

Your prephrase is that the correct answer choice will describe the astronomer accepting the view of the two biologists over that of Tagar. Although you might be tempted to think that because the astronomer chooses the two biologists over the solitary voice of Tagar that this is an appeal to numbers flaw. However, the argument makes no reference to the numbers idea, and so that is unlikely to be the correct answer. Instead, your prephrase should focus on the lack of any support for the astronomer’s preference.

Answer choice (A): This choice is incorrect because the astronomer’s argument makes no reference to the consistency of the views held by the biologists. This choice is a type of source argument, in which the inconsistency in a person’s view is held out as evidence that their more recent view is incorrect.

Answer choice (B): This is the correct answer choice. This choice points to the lack of evidence to support the astronomer’s implicit preference for the view of Swiderski and Terrada over that of Tagar.

Answer choice (C): This choice is incorrect, because the astronomer makes no mention of the difference in number, and an appeal to numbers is not inherently the basis of his conclusion.

Answer choice (D): This choice describes an internally inconsistent argument, which was not present in the stimulus.

Answer choice (E): This choice describes the opposite of what was present in the stimulus, in which the astronomer preferred the opinion of two apparent experts over that of another.
 rneuman123@gmail.com
  • Posts: 38
  • Joined: Aug 17, 2016
|
#28457
This question really threw me off. I was going to choose b, but it says: "The argument provides no justification for giving preference to the views of ONE RATHER THAN THE OTHER TWO COMPETING SETS OF AUTHORITIES." But the stimulus didn't do that. The astronomer agreed with the other scientists over the professor. It seems to me that B switches it around.
User avatar
 Jonathan Evans
PowerScore Staff
  • PowerScore Staff
  • Posts: 726
  • Joined: Jun 09, 2016
|
#28480
Rneuman,

Yeah, what can I say, this stinks, but you missed a word in the answer choice.

The answer reads:

"The argument provides no justification for giving preference to the views of one rather than the other of two competing sets of authorities."

The sets of authorities are delineated by the view espoused by Tagar versus that of Swiderski and Terrada. The word "of" separates the preceding prepositional phrase from the following prepositional phrase.

It stinks, but that's classic LSAT.
 Pragmatism
  • Posts: 68
  • Joined: Jan 11, 2018
|
#43706
I selected the the right answer. However, it was my understanding that when a flaw method of reasoning questions is presented, the answer choice should encompass all the flaws within that stimulus. While it is true that the astronomer doesn't give us a reason as to why the one side is right and the other wrong, I saw another inherent flaw which is what I would like you to comment on. I would like to know whether you would constitute this as a flaw along with the flaw mentioned in the answer choice.

So, Tagar say's, it cannot be fossilized bacteria. Why? Because they are 1/10 of 1% of the smallest earthly bacteria. However, Tagar distorts Swiderski's and Terrada's initial premise, which was that, "bacteria would shrink to 1/10 of 1% of their normal volume when water or other nutrients were in short supply." And if that is a flaw worthy of note, then could there be a case that the astronomer, while not giving much support other than the premise for the conclusion, is stating that the flaw committed by Tagar is sufficient enough to disagree with Professor Tagar, and the flaw that was committed in the right answer choice isn't the flaw that this stimulus wanted to showcase?
 Claire Horan
PowerScore Staff
  • PowerScore Staff
  • Posts: 408
  • Joined: Apr 18, 2016
|
#43733
Hi pragmatism,

What you are pointing to isn't a flaw in the argument at all. Tagar doesn't "distort Swiderski's and Terrada's initial premise"—the argument doesn't show Tagar responding the Swiderski and Terrada at all. Instead, they respond to his claim.

Tagar doesn't think it's possible that the bacteria-like structures are bacteria because they are so much smaller than the very smallest bacteria found on Earth. Swiderski and Terrada disagree because they believe (without any noted justification) that the normal volume of the Martian bacteria could have been as large as those found on Earth but they would shrink to 1/10 of one percent of their normal volume as fossils (which would tend to show that their normal volume could be similar to the normal volume of Earth bacteria).
 Khodi7531
  • Posts: 116
  • Joined: Mar 14, 2018
|
#44798
I got to B after I realized everything else sucks but then understood what it was saying.

Once I thought for a second I spotted the issue as the author/argument is saying Tagar's grounds are wrong because others have said that same evidence can prove something. But it never establishes that maybe Tagar conclusively disagrees with the other biologists view point for a reason, and that's why he declared the meteorite to not be fossilized.

So I saw B as attacking that gap of, "he never addresses the views of Tagars reasoning for why he doesn't accept their view"


Is that correct? Reading everything else in this forum got me confused lol.
 Francis O'Rourke
PowerScore Staff
  • PowerScore Staff
  • Posts: 471
  • Joined: Mar 10, 2017
|
#44926
it never establishes that maybe Tagar conclusively disagrees with the other biologists view point for a reason,
This sounds like another way of expressing the flaw that answer choice (B) points out: Tagar never gives us a good reason for supporting his view over the two other biologists' views.

If there was any specific aspect of the discussion above that confused you though, let me know and I will try to address it for you! :-D
 Blueballoon5%
  • Posts: 156
  • Joined: Jul 13, 2015
|
#50041
I got this question wrong because I misinterpreted the wording of answer choice D (I think). When reading this answer choice, I thought it meant that the argument appealed to the two views (Tagar's and the other two biologists) that contradict each other, and they did that in the stimulus. I think my main mistake was the interpretation of the word "appeal" in this answer choice. In the LSAT, what does "appeal" mean (e.g. "I appeal to a certain authority")?
 Brook Miscoski
PowerScore Staff
  • PowerScore Staff
  • Posts: 418
  • Joined: Sep 13, 2018
|
#62677
Blueballoon,

In context "appeals to" basically means "uses as support." The problem with (D) is not the word "appeals." Instead, the problem with (D) is that the stimulus does not use contradictory viewpoints for support. The argument presents contradictory viewpoints and takes a side.
 Boudreaux
  • Posts: 9
  • Joined: Jan 04, 2019
|
#62694
I understand the wording in B that states "the argument provides no justification," but I was hung up on considering C for some time and I wanted to ask this. The wording in answer choice C states "the argument takes for granted that" and I interpret that statement to read there must be conditional reasoning for that to be true. Ultimately that is why ruled C out, but I went back to the stimulus and searched for conditional reasoning that I could not find. Would you please comment?

And just for clarity, I "think" provides no justification means no direct link between premise and conclusion

Thanks

Get the most out of your LSAT Prep Plus subscription.

Analyze and track your performance with our Testing and Analytics Package.